Eigenvalues of MSome properties of a $2times 2$ matrix with repeated eigenvaluesAnalogy between Linear Transformations & MatricesLinear algeba - eigenvalues and zero matrixJacobian Eigenvalue Algorithm and Positive definiteness of Eigenvalue matrixPick the true options for a $ntimes n$ matrix $A$Let $A$ be an $ntimes n$ matrices over $mathbbC$ then which of the following are true?How to find size of each Jordan BlockShow that every integer eigenvalue of $A$ divides the determinant of $A$.How Eigenvectors of $A^T$ are perpendicular to eigen vectors of A?Eigenvalues of a square matrix

Greatest common substring

Books on the History of math research at European universities

My boss asked me to take a one-day class, then signs it up as a day off

"lassen" in meaning "sich fassen"

Organic chemistry Iodoform Reaction

Why are on-board computers allowed to change controls without notifying the pilots?

Does "Dominei" mean something?

How to check participants in at events?

Identify a stage play about a VR experience in which participants are encouraged to simulate performing horrific activities

Simple image editor tool to draw a simple box/rectangle in an existing image

Adding empty element to declared container without declaring type of element

How will losing mobility of one hand affect my career as a programmer?

Can a Gentile theist be saved?

Science Fiction story where a man invents a machine that can help him watch history unfold

Hostile work environment after whistle-blowing on coworker and our boss. What do I do?

Would it be legal for a US State to ban exports of a natural resource?

How can I successfully establish a nationwide combat training program for a large country?

What is the opposite of 'gravitas'?

Proof of Lemma: Every integer can be written as a product of primes

Meta programming: Declare a new struct on the fly

The One-Electron Universe postulate is true - what simple change can I make to change the whole universe?

Latex for-and in equation

Can somebody explain Brexit in a few child-proof sentences?

Is there enough fresh water in the world to eradicate the drinking water crisis?



Eigenvalues of M


Some properties of a $2times 2$ matrix with repeated eigenvaluesAnalogy between Linear Transformations & MatricesLinear algeba - eigenvalues and zero matrixJacobian Eigenvalue Algorithm and Positive definiteness of Eigenvalue matrixPick the true options for a $ntimes n$ matrix $A$Let $A$ be an $ntimes n$ matrices over $mathbbC$ then which of the following are true?How to find size of each Jordan BlockShow that every integer eigenvalue of $A$ divides the determinant of $A$.How Eigenvectors of $A^T$ are perpendicular to eigen vectors of A?Eigenvalues of a square matrix













1












$begingroup$


Let M be square matrix of order n with real entries Satisfying $M^3=I$ and $Mvneq v$ for any non-zero vector $v$.Then Which of the followings are true?



$1.$ M has real eigen-values?



$2.$ $M+M^-1$ has real eigen-values?



$3.n$ is divisible by 2.



$4.n$ is divisible by 3.



$Mvneq v$ for any non-zero vector $v$ means 1 is not an eigenvalue of M.Hence $x^2+x+1=0$ is minimal polynomial of M.Hence 2 is correct option.But I am not able to understand why 1st and 4th options are correct?










share|cite|improve this question









$endgroup$







  • 1




    $begingroup$
    The first statement definitely cannot be true, since if $lambda$ is an eigenvalue of $M$ then $Mv= lambda v implies v = lambda^3 v$ so $lambda^3 = 1$ but $lambda neq 1$ so $lambda$ is necessarily complex. The second is right since $M + M^-1 = -I$.
    $endgroup$
    – астон вілла олоф мэллбэрг
    Jun 15 '18 at 6:08
















1












$begingroup$


Let M be square matrix of order n with real entries Satisfying $M^3=I$ and $Mvneq v$ for any non-zero vector $v$.Then Which of the followings are true?



$1.$ M has real eigen-values?



$2.$ $M+M^-1$ has real eigen-values?



$3.n$ is divisible by 2.



$4.n$ is divisible by 3.



$Mvneq v$ for any non-zero vector $v$ means 1 is not an eigenvalue of M.Hence $x^2+x+1=0$ is minimal polynomial of M.Hence 2 is correct option.But I am not able to understand why 1st and 4th options are correct?










share|cite|improve this question









$endgroup$







  • 1




    $begingroup$
    The first statement definitely cannot be true, since if $lambda$ is an eigenvalue of $M$ then $Mv= lambda v implies v = lambda^3 v$ so $lambda^3 = 1$ but $lambda neq 1$ so $lambda$ is necessarily complex. The second is right since $M + M^-1 = -I$.
    $endgroup$
    – астон вілла олоф мэллбэрг
    Jun 15 '18 at 6:08














1












1








1


1



$begingroup$


Let M be square matrix of order n with real entries Satisfying $M^3=I$ and $Mvneq v$ for any non-zero vector $v$.Then Which of the followings are true?



$1.$ M has real eigen-values?



$2.$ $M+M^-1$ has real eigen-values?



$3.n$ is divisible by 2.



$4.n$ is divisible by 3.



$Mvneq v$ for any non-zero vector $v$ means 1 is not an eigenvalue of M.Hence $x^2+x+1=0$ is minimal polynomial of M.Hence 2 is correct option.But I am not able to understand why 1st and 4th options are correct?










share|cite|improve this question









$endgroup$




Let M be square matrix of order n with real entries Satisfying $M^3=I$ and $Mvneq v$ for any non-zero vector $v$.Then Which of the followings are true?



$1.$ M has real eigen-values?



$2.$ $M+M^-1$ has real eigen-values?



$3.n$ is divisible by 2.



$4.n$ is divisible by 3.



$Mvneq v$ for any non-zero vector $v$ means 1 is not an eigenvalue of M.Hence $x^2+x+1=0$ is minimal polynomial of M.Hence 2 is correct option.But I am not able to understand why 1st and 4th options are correct?







linear-algebra eigenvalues-eigenvectors






share|cite|improve this question













share|cite|improve this question











share|cite|improve this question




share|cite|improve this question










asked Jun 15 '18 at 5:35









ASHWINI SANKHEASHWINI SANKHE

12710




12710







  • 1




    $begingroup$
    The first statement definitely cannot be true, since if $lambda$ is an eigenvalue of $M$ then $Mv= lambda v implies v = lambda^3 v$ so $lambda^3 = 1$ but $lambda neq 1$ so $lambda$ is necessarily complex. The second is right since $M + M^-1 = -I$.
    $endgroup$
    – астон вілла олоф мэллбэрг
    Jun 15 '18 at 6:08













  • 1




    $begingroup$
    The first statement definitely cannot be true, since if $lambda$ is an eigenvalue of $M$ then $Mv= lambda v implies v = lambda^3 v$ so $lambda^3 = 1$ but $lambda neq 1$ so $lambda$ is necessarily complex. The second is right since $M + M^-1 = -I$.
    $endgroup$
    – астон вілла олоф мэллбэрг
    Jun 15 '18 at 6:08








1




1




$begingroup$
The first statement definitely cannot be true, since if $lambda$ is an eigenvalue of $M$ then $Mv= lambda v implies v = lambda^3 v$ so $lambda^3 = 1$ but $lambda neq 1$ so $lambda$ is necessarily complex. The second is right since $M + M^-1 = -I$.
$endgroup$
– астон вілла олоф мэллбэрг
Jun 15 '18 at 6:08





$begingroup$
The first statement definitely cannot be true, since if $lambda$ is an eigenvalue of $M$ then $Mv= lambda v implies v = lambda^3 v$ so $lambda^3 = 1$ but $lambda neq 1$ so $lambda$ is necessarily complex. The second is right since $M + M^-1 = -I$.
$endgroup$
– астон вілла олоф мэллбэрг
Jun 15 '18 at 6:08











1 Answer
1






active

oldest

votes


















2












$begingroup$

  1. $M^3=I$ so $M^3-I=(M-I)(M^2+M+I)=0$ but for hypothesis you have that $M-I $ is invertibile and so:
    $M^2+M+I=0$
    By contraddiction if there exists a real eighenvalue $lambda$ of M then , if $vneq0$ is his eighenvector, you have that
    $(M^2+M+I)v=(lambda^2+lambda+1)v=0$
    and so
    $lambda^2+lambda+1=0$
    But this polinomial equation not have solutions in $mathbbR$.


  2. $M^2+M+I=0$ and you can multiply both members for $M^-1$ :


$M+I+M^-1=0$



So



$M+M^-1=-I $
that is diagonalizzable oviously.



  1. What you said is correct.

By contraddiction if $ n=deg(det(M-lambda I) $ is not divisibile for 2 then the polinomial have necessary a real root because any polinomial of degree odd have al least a root in $mathbbR$



  1. It is false because there exists a $2x2$- real matrix $M$ such that $M^3=I$ and $M-I$ is invertibile. An example can be

$beginbmatrix-frac12&fracsqrt32\ -fracsqrt32&-frac12endbmatrix$






share|cite|improve this answer











$endgroup$












    Your Answer





    StackExchange.ifUsing("editor", function ()
    return StackExchange.using("mathjaxEditing", function ()
    StackExchange.MarkdownEditor.creationCallbacks.add(function (editor, postfix)
    StackExchange.mathjaxEditing.prepareWmdForMathJax(editor, postfix, [["$", "$"], ["\\(","\\)"]]);
    );
    );
    , "mathjax-editing");

    StackExchange.ready(function()
    var channelOptions =
    tags: "".split(" "),
    id: "69"
    ;
    initTagRenderer("".split(" "), "".split(" "), channelOptions);

    StackExchange.using("externalEditor", function()
    // Have to fire editor after snippets, if snippets enabled
    if (StackExchange.settings.snippets.snippetsEnabled)
    StackExchange.using("snippets", function()
    createEditor();
    );

    else
    createEditor();

    );

    function createEditor()
    StackExchange.prepareEditor(
    heartbeatType: 'answer',
    autoActivateHeartbeat: false,
    convertImagesToLinks: true,
    noModals: true,
    showLowRepImageUploadWarning: true,
    reputationToPostImages: 10,
    bindNavPrevention: true,
    postfix: "",
    imageUploader:
    brandingHtml: "Powered by u003ca class="icon-imgur-white" href="https://imgur.com/"u003eu003c/au003e",
    contentPolicyHtml: "User contributions licensed under u003ca href="https://creativecommons.org/licenses/by-sa/3.0/"u003ecc by-sa 3.0 with attribution requiredu003c/au003e u003ca href="https://stackoverflow.com/legal/content-policy"u003e(content policy)u003c/au003e",
    allowUrls: true
    ,
    noCode: true, onDemand: true,
    discardSelector: ".discard-answer"
    ,immediatelyShowMarkdownHelp:true
    );



    );













    draft saved

    draft discarded


















    StackExchange.ready(
    function ()
    StackExchange.openid.initPostLogin('.new-post-login', 'https%3a%2f%2fmath.stackexchange.com%2fquestions%2f2820323%2feigenvalues-of-m%23new-answer', 'question_page');

    );

    Post as a guest















    Required, but never shown

























    1 Answer
    1






    active

    oldest

    votes








    1 Answer
    1






    active

    oldest

    votes









    active

    oldest

    votes






    active

    oldest

    votes









    2












    $begingroup$

    1. $M^3=I$ so $M^3-I=(M-I)(M^2+M+I)=0$ but for hypothesis you have that $M-I $ is invertibile and so:
      $M^2+M+I=0$
      By contraddiction if there exists a real eighenvalue $lambda$ of M then , if $vneq0$ is his eighenvector, you have that
      $(M^2+M+I)v=(lambda^2+lambda+1)v=0$
      and so
      $lambda^2+lambda+1=0$
      But this polinomial equation not have solutions in $mathbbR$.


    2. $M^2+M+I=0$ and you can multiply both members for $M^-1$ :


    $M+I+M^-1=0$



    So



    $M+M^-1=-I $
    that is diagonalizzable oviously.



    1. What you said is correct.

    By contraddiction if $ n=deg(det(M-lambda I) $ is not divisibile for 2 then the polinomial have necessary a real root because any polinomial of degree odd have al least a root in $mathbbR$



    1. It is false because there exists a $2x2$- real matrix $M$ such that $M^3=I$ and $M-I$ is invertibile. An example can be

    $beginbmatrix-frac12&fracsqrt32\ -fracsqrt32&-frac12endbmatrix$






    share|cite|improve this answer











    $endgroup$

















      2












      $begingroup$

      1. $M^3=I$ so $M^3-I=(M-I)(M^2+M+I)=0$ but for hypothesis you have that $M-I $ is invertibile and so:
        $M^2+M+I=0$
        By contraddiction if there exists a real eighenvalue $lambda$ of M then , if $vneq0$ is his eighenvector, you have that
        $(M^2+M+I)v=(lambda^2+lambda+1)v=0$
        and so
        $lambda^2+lambda+1=0$
        But this polinomial equation not have solutions in $mathbbR$.


      2. $M^2+M+I=0$ and you can multiply both members for $M^-1$ :


      $M+I+M^-1=0$



      So



      $M+M^-1=-I $
      that is diagonalizzable oviously.



      1. What you said is correct.

      By contraddiction if $ n=deg(det(M-lambda I) $ is not divisibile for 2 then the polinomial have necessary a real root because any polinomial of degree odd have al least a root in $mathbbR$



      1. It is false because there exists a $2x2$- real matrix $M$ such that $M^3=I$ and $M-I$ is invertibile. An example can be

      $beginbmatrix-frac12&fracsqrt32\ -fracsqrt32&-frac12endbmatrix$






      share|cite|improve this answer











      $endgroup$















        2












        2








        2





        $begingroup$

        1. $M^3=I$ so $M^3-I=(M-I)(M^2+M+I)=0$ but for hypothesis you have that $M-I $ is invertibile and so:
          $M^2+M+I=0$
          By contraddiction if there exists a real eighenvalue $lambda$ of M then , if $vneq0$ is his eighenvector, you have that
          $(M^2+M+I)v=(lambda^2+lambda+1)v=0$
          and so
          $lambda^2+lambda+1=0$
          But this polinomial equation not have solutions in $mathbbR$.


        2. $M^2+M+I=0$ and you can multiply both members for $M^-1$ :


        $M+I+M^-1=0$



        So



        $M+M^-1=-I $
        that is diagonalizzable oviously.



        1. What you said is correct.

        By contraddiction if $ n=deg(det(M-lambda I) $ is not divisibile for 2 then the polinomial have necessary a real root because any polinomial of degree odd have al least a root in $mathbbR$



        1. It is false because there exists a $2x2$- real matrix $M$ such that $M^3=I$ and $M-I$ is invertibile. An example can be

        $beginbmatrix-frac12&fracsqrt32\ -fracsqrt32&-frac12endbmatrix$






        share|cite|improve this answer











        $endgroup$



        1. $M^3=I$ so $M^3-I=(M-I)(M^2+M+I)=0$ but for hypothesis you have that $M-I $ is invertibile and so:
          $M^2+M+I=0$
          By contraddiction if there exists a real eighenvalue $lambda$ of M then , if $vneq0$ is his eighenvector, you have that
          $(M^2+M+I)v=(lambda^2+lambda+1)v=0$
          and so
          $lambda^2+lambda+1=0$
          But this polinomial equation not have solutions in $mathbbR$.


        2. $M^2+M+I=0$ and you can multiply both members for $M^-1$ :


        $M+I+M^-1=0$



        So



        $M+M^-1=-I $
        that is diagonalizzable oviously.



        1. What you said is correct.

        By contraddiction if $ n=deg(det(M-lambda I) $ is not divisibile for 2 then the polinomial have necessary a real root because any polinomial of degree odd have al least a root in $mathbbR$



        1. It is false because there exists a $2x2$- real matrix $M$ such that $M^3=I$ and $M-I$ is invertibile. An example can be

        $beginbmatrix-frac12&fracsqrt32\ -fracsqrt32&-frac12endbmatrix$







        share|cite|improve this answer














        share|cite|improve this answer



        share|cite|improve this answer








        edited Mar 16 at 20:42

























        answered Jun 15 '18 at 10:38









        Federico FalluccaFederico Fallucca

        2,280210




        2,280210



























            draft saved

            draft discarded
















































            Thanks for contributing an answer to Mathematics Stack Exchange!


            • Please be sure to answer the question. Provide details and share your research!

            But avoid


            • Asking for help, clarification, or responding to other answers.

            • Making statements based on opinion; back them up with references or personal experience.

            Use MathJax to format equations. MathJax reference.


            To learn more, see our tips on writing great answers.




            draft saved


            draft discarded














            StackExchange.ready(
            function ()
            StackExchange.openid.initPostLogin('.new-post-login', 'https%3a%2f%2fmath.stackexchange.com%2fquestions%2f2820323%2feigenvalues-of-m%23new-answer', 'question_page');

            );

            Post as a guest















            Required, but never shown





















































            Required, but never shown














            Required, but never shown












            Required, but never shown







            Required, but never shown

































            Required, but never shown














            Required, but never shown












            Required, but never shown







            Required, but never shown







            Popular posts from this blog

            Lowndes Grove History Architecture References Navigation menu32°48′6″N 79°57′58″W / 32.80167°N 79.96611°W / 32.80167; -79.9661132°48′6″N 79°57′58″W / 32.80167°N 79.96611°W / 32.80167; -79.9661178002500"National Register Information System"Historic houses of South Carolina"Lowndes Grove""+32° 48' 6.00", −79° 57' 58.00""Lowndes Grove, Charleston County (260 St. Margaret St., Charleston)""Lowndes Grove"The Charleston ExpositionIt Happened in South Carolina"Lowndes Grove (House), Saint Margaret Street & Sixth Avenue, Charleston, Charleston County, SC(Photographs)"Plantations of the Carolina Low Countrye

            random experiment with two different functions on unit interval Announcing the arrival of Valued Associate #679: Cesar Manara Planned maintenance scheduled April 23, 2019 at 00:00UTC (8:00pm US/Eastern)Random variable and probability space notionsRandom Walk with EdgesFinding functions where the increase over a random interval is Poisson distributedNumber of days until dayCan an observed event in fact be of zero probability?Unit random processmodels of coins and uniform distributionHow to get the number of successes given $n$ trials , probability $P$ and a random variable $X$Absorbing Markov chain in a computer. Is “almost every” turned into always convergence in computer executions?Stopped random walk is not uniformly integrable

            How should I support this large drywall patch? Planned maintenance scheduled April 23, 2019 at 00:00UTC (8:00pm US/Eastern) Announcing the arrival of Valued Associate #679: Cesar Manara Unicorn Meta Zoo #1: Why another podcast?How do I cover large gaps in drywall?How do I keep drywall around a patch from crumbling?Can I glue a second layer of drywall?How to patch long strip on drywall?Large drywall patch: how to avoid bulging seams?Drywall Mesh Patch vs. Bulge? To remove or not to remove?How to fix this drywall job?Prep drywall before backsplashWhat's the best way to fix this horrible drywall patch job?Drywall patching using 3M Patch Plus Primer